Is there a way to sum (part of) the harmonic series to a given total?












0












$begingroup$


For example, $1 + 1/2 + 1/3 + 1/4 + 1/5 + 1/6$ can be solved by traditional arithmetic of $1 + 360/720 + 240/720...$ etc. to get a total of $2.45$, but if I wanted the total to say, $1/12367$, the value of which sum is about $10$, is there a fast process for acquiring this total? I'm hoping there's something similar to Gauss's solution to arithmetic sequences.










share|cite|improve this question











$endgroup$












  • $begingroup$
    No chance for an exact formula. There are very sharp estimates though.
    $endgroup$
    – A. Pongrácz
    Jan 2 at 23:47










  • $begingroup$
    Also, this looks like a duplicate. See here
    $endgroup$
    – jmerry
    Jan 2 at 23:48
















0












$begingroup$


For example, $1 + 1/2 + 1/3 + 1/4 + 1/5 + 1/6$ can be solved by traditional arithmetic of $1 + 360/720 + 240/720...$ etc. to get a total of $2.45$, but if I wanted the total to say, $1/12367$, the value of which sum is about $10$, is there a fast process for acquiring this total? I'm hoping there's something similar to Gauss's solution to arithmetic sequences.










share|cite|improve this question











$endgroup$












  • $begingroup$
    No chance for an exact formula. There are very sharp estimates though.
    $endgroup$
    – A. Pongrácz
    Jan 2 at 23:47










  • $begingroup$
    Also, this looks like a duplicate. See here
    $endgroup$
    – jmerry
    Jan 2 at 23:48














0












0








0





$begingroup$


For example, $1 + 1/2 + 1/3 + 1/4 + 1/5 + 1/6$ can be solved by traditional arithmetic of $1 + 360/720 + 240/720...$ etc. to get a total of $2.45$, but if I wanted the total to say, $1/12367$, the value of which sum is about $10$, is there a fast process for acquiring this total? I'm hoping there's something similar to Gauss's solution to arithmetic sequences.










share|cite|improve this question











$endgroup$




For example, $1 + 1/2 + 1/3 + 1/4 + 1/5 + 1/6$ can be solved by traditional arithmetic of $1 + 360/720 + 240/720...$ etc. to get a total of $2.45$, but if I wanted the total to say, $1/12367$, the value of which sum is about $10$, is there a fast process for acquiring this total? I'm hoping there's something similar to Gauss's solution to arithmetic sequences.







sequences-and-series summation harmonic-numbers






share|cite|improve this question















share|cite|improve this question













share|cite|improve this question




share|cite|improve this question








edited Jan 3 at 0:12









Eevee Trainer

7,04321337




7,04321337










asked Jan 2 at 23:41









TristianTristian

1769




1769












  • $begingroup$
    No chance for an exact formula. There are very sharp estimates though.
    $endgroup$
    – A. Pongrácz
    Jan 2 at 23:47










  • $begingroup$
    Also, this looks like a duplicate. See here
    $endgroup$
    – jmerry
    Jan 2 at 23:48


















  • $begingroup$
    No chance for an exact formula. There are very sharp estimates though.
    $endgroup$
    – A. Pongrácz
    Jan 2 at 23:47










  • $begingroup$
    Also, this looks like a duplicate. See here
    $endgroup$
    – jmerry
    Jan 2 at 23:48
















$begingroup$
No chance for an exact formula. There are very sharp estimates though.
$endgroup$
– A. Pongrácz
Jan 2 at 23:47




$begingroup$
No chance for an exact formula. There are very sharp estimates though.
$endgroup$
– A. Pongrácz
Jan 2 at 23:47












$begingroup$
Also, this looks like a duplicate. See here
$endgroup$
– jmerry
Jan 2 at 23:48




$begingroup$
Also, this looks like a duplicate. See here
$endgroup$
– jmerry
Jan 2 at 23:48










2 Answers
2






active

oldest

votes


















2












$begingroup$

There is a sort of way to sum the series, but it's not exact. $$1 + frac{1}{2} + frac{1}{3} + frac{1}{4} + ... + frac{1}{n} = sum_{k=1}^n frac{1}{k} = ln(n) + gamma + epsilon_n$$



$epsilon_n$ is an error constant proportional to $1/2n$, and thus $epsilon_n to 0$ as $n to infty$.



$gamma$ denotes the Euler-Mascheroni constant, defined by the limiting difference between the natural logarithm and the harmonic series, i.e.



$${displaystyle {begin{aligned}gamma &=lim _{nto infty }left(-ln n+sum _{k=1}^{n}{frac {1}{k}}right)\[5px]end{aligned}}} approx 0.5772156649$$



Thus, you can approximate the sum of the first $n$ natural reciprocals by $ln(n)$, plus $gamma$ if you so choose, with greater accuracy as $n to infty$. However, there is no known exact formula for these partial sums.






share|cite|improve this answer









$endgroup$





















    2












    $begingroup$

    Eevee Trainer gave the right idea that is to say considering the asymptotics of harmonic numbers.



    If you want more accuracy, you could use
    $$ sum_{k=1}^n frac{1}{k}=H_n=gamma +log left({n}right)+frac{1}{2 n}-frac{1}{12
    n^2}+Oleft(frac{1}{n^4}right)$$



    Trying with $n=12367$, the "exact" value is $10.00004300827580769471$ while the above approximation would give $10.00004300827580769435$






    share|cite|improve this answer









    $endgroup$













      Your Answer





      StackExchange.ifUsing("editor", function () {
      return StackExchange.using("mathjaxEditing", function () {
      StackExchange.MarkdownEditor.creationCallbacks.add(function (editor, postfix) {
      StackExchange.mathjaxEditing.prepareWmdForMathJax(editor, postfix, [["$", "$"], ["\\(","\\)"]]);
      });
      });
      }, "mathjax-editing");

      StackExchange.ready(function() {
      var channelOptions = {
      tags: "".split(" "),
      id: "69"
      };
      initTagRenderer("".split(" "), "".split(" "), channelOptions);

      StackExchange.using("externalEditor", function() {
      // Have to fire editor after snippets, if snippets enabled
      if (StackExchange.settings.snippets.snippetsEnabled) {
      StackExchange.using("snippets", function() {
      createEditor();
      });
      }
      else {
      createEditor();
      }
      });

      function createEditor() {
      StackExchange.prepareEditor({
      heartbeatType: 'answer',
      autoActivateHeartbeat: false,
      convertImagesToLinks: true,
      noModals: true,
      showLowRepImageUploadWarning: true,
      reputationToPostImages: 10,
      bindNavPrevention: true,
      postfix: "",
      imageUploader: {
      brandingHtml: "Powered by u003ca class="icon-imgur-white" href="https://imgur.com/"u003eu003c/au003e",
      contentPolicyHtml: "User contributions licensed under u003ca href="https://creativecommons.org/licenses/by-sa/3.0/"u003ecc by-sa 3.0 with attribution requiredu003c/au003e u003ca href="https://stackoverflow.com/legal/content-policy"u003e(content policy)u003c/au003e",
      allowUrls: true
      },
      noCode: true, onDemand: true,
      discardSelector: ".discard-answer"
      ,immediatelyShowMarkdownHelp:true
      });


      }
      });














      draft saved

      draft discarded


















      StackExchange.ready(
      function () {
      StackExchange.openid.initPostLogin('.new-post-login', 'https%3a%2f%2fmath.stackexchange.com%2fquestions%2f3060107%2fis-there-a-way-to-sum-part-of-the-harmonic-series-to-a-given-total%23new-answer', 'question_page');
      }
      );

      Post as a guest















      Required, but never shown

























      2 Answers
      2






      active

      oldest

      votes








      2 Answers
      2






      active

      oldest

      votes









      active

      oldest

      votes






      active

      oldest

      votes









      2












      $begingroup$

      There is a sort of way to sum the series, but it's not exact. $$1 + frac{1}{2} + frac{1}{3} + frac{1}{4} + ... + frac{1}{n} = sum_{k=1}^n frac{1}{k} = ln(n) + gamma + epsilon_n$$



      $epsilon_n$ is an error constant proportional to $1/2n$, and thus $epsilon_n to 0$ as $n to infty$.



      $gamma$ denotes the Euler-Mascheroni constant, defined by the limiting difference between the natural logarithm and the harmonic series, i.e.



      $${displaystyle {begin{aligned}gamma &=lim _{nto infty }left(-ln n+sum _{k=1}^{n}{frac {1}{k}}right)\[5px]end{aligned}}} approx 0.5772156649$$



      Thus, you can approximate the sum of the first $n$ natural reciprocals by $ln(n)$, plus $gamma$ if you so choose, with greater accuracy as $n to infty$. However, there is no known exact formula for these partial sums.






      share|cite|improve this answer









      $endgroup$


















        2












        $begingroup$

        There is a sort of way to sum the series, but it's not exact. $$1 + frac{1}{2} + frac{1}{3} + frac{1}{4} + ... + frac{1}{n} = sum_{k=1}^n frac{1}{k} = ln(n) + gamma + epsilon_n$$



        $epsilon_n$ is an error constant proportional to $1/2n$, and thus $epsilon_n to 0$ as $n to infty$.



        $gamma$ denotes the Euler-Mascheroni constant, defined by the limiting difference between the natural logarithm and the harmonic series, i.e.



        $${displaystyle {begin{aligned}gamma &=lim _{nto infty }left(-ln n+sum _{k=1}^{n}{frac {1}{k}}right)\[5px]end{aligned}}} approx 0.5772156649$$



        Thus, you can approximate the sum of the first $n$ natural reciprocals by $ln(n)$, plus $gamma$ if you so choose, with greater accuracy as $n to infty$. However, there is no known exact formula for these partial sums.






        share|cite|improve this answer









        $endgroup$
















          2












          2








          2





          $begingroup$

          There is a sort of way to sum the series, but it's not exact. $$1 + frac{1}{2} + frac{1}{3} + frac{1}{4} + ... + frac{1}{n} = sum_{k=1}^n frac{1}{k} = ln(n) + gamma + epsilon_n$$



          $epsilon_n$ is an error constant proportional to $1/2n$, and thus $epsilon_n to 0$ as $n to infty$.



          $gamma$ denotes the Euler-Mascheroni constant, defined by the limiting difference between the natural logarithm and the harmonic series, i.e.



          $${displaystyle {begin{aligned}gamma &=lim _{nto infty }left(-ln n+sum _{k=1}^{n}{frac {1}{k}}right)\[5px]end{aligned}}} approx 0.5772156649$$



          Thus, you can approximate the sum of the first $n$ natural reciprocals by $ln(n)$, plus $gamma$ if you so choose, with greater accuracy as $n to infty$. However, there is no known exact formula for these partial sums.






          share|cite|improve this answer









          $endgroup$



          There is a sort of way to sum the series, but it's not exact. $$1 + frac{1}{2} + frac{1}{3} + frac{1}{4} + ... + frac{1}{n} = sum_{k=1}^n frac{1}{k} = ln(n) + gamma + epsilon_n$$



          $epsilon_n$ is an error constant proportional to $1/2n$, and thus $epsilon_n to 0$ as $n to infty$.



          $gamma$ denotes the Euler-Mascheroni constant, defined by the limiting difference between the natural logarithm and the harmonic series, i.e.



          $${displaystyle {begin{aligned}gamma &=lim _{nto infty }left(-ln n+sum _{k=1}^{n}{frac {1}{k}}right)\[5px]end{aligned}}} approx 0.5772156649$$



          Thus, you can approximate the sum of the first $n$ natural reciprocals by $ln(n)$, plus $gamma$ if you so choose, with greater accuracy as $n to infty$. However, there is no known exact formula for these partial sums.







          share|cite|improve this answer












          share|cite|improve this answer



          share|cite|improve this answer










          answered Jan 3 at 0:09









          Eevee TrainerEevee Trainer

          7,04321337




          7,04321337























              2












              $begingroup$

              Eevee Trainer gave the right idea that is to say considering the asymptotics of harmonic numbers.



              If you want more accuracy, you could use
              $$ sum_{k=1}^n frac{1}{k}=H_n=gamma +log left({n}right)+frac{1}{2 n}-frac{1}{12
              n^2}+Oleft(frac{1}{n^4}right)$$



              Trying with $n=12367$, the "exact" value is $10.00004300827580769471$ while the above approximation would give $10.00004300827580769435$






              share|cite|improve this answer









              $endgroup$


















                2












                $begingroup$

                Eevee Trainer gave the right idea that is to say considering the asymptotics of harmonic numbers.



                If you want more accuracy, you could use
                $$ sum_{k=1}^n frac{1}{k}=H_n=gamma +log left({n}right)+frac{1}{2 n}-frac{1}{12
                n^2}+Oleft(frac{1}{n^4}right)$$



                Trying with $n=12367$, the "exact" value is $10.00004300827580769471$ while the above approximation would give $10.00004300827580769435$






                share|cite|improve this answer









                $endgroup$
















                  2












                  2








                  2





                  $begingroup$

                  Eevee Trainer gave the right idea that is to say considering the asymptotics of harmonic numbers.



                  If you want more accuracy, you could use
                  $$ sum_{k=1}^n frac{1}{k}=H_n=gamma +log left({n}right)+frac{1}{2 n}-frac{1}{12
                  n^2}+Oleft(frac{1}{n^4}right)$$



                  Trying with $n=12367$, the "exact" value is $10.00004300827580769471$ while the above approximation would give $10.00004300827580769435$






                  share|cite|improve this answer









                  $endgroup$



                  Eevee Trainer gave the right idea that is to say considering the asymptotics of harmonic numbers.



                  If you want more accuracy, you could use
                  $$ sum_{k=1}^n frac{1}{k}=H_n=gamma +log left({n}right)+frac{1}{2 n}-frac{1}{12
                  n^2}+Oleft(frac{1}{n^4}right)$$



                  Trying with $n=12367$, the "exact" value is $10.00004300827580769471$ while the above approximation would give $10.00004300827580769435$







                  share|cite|improve this answer












                  share|cite|improve this answer



                  share|cite|improve this answer










                  answered Jan 3 at 9:42









                  Claude LeiboviciClaude Leibovici

                  123k1157134




                  123k1157134






























                      draft saved

                      draft discarded




















































                      Thanks for contributing an answer to Mathematics Stack Exchange!


                      • Please be sure to answer the question. Provide details and share your research!

                      But avoid



                      • Asking for help, clarification, or responding to other answers.

                      • Making statements based on opinion; back them up with references or personal experience.


                      Use MathJax to format equations. MathJax reference.


                      To learn more, see our tips on writing great answers.




                      draft saved


                      draft discarded














                      StackExchange.ready(
                      function () {
                      StackExchange.openid.initPostLogin('.new-post-login', 'https%3a%2f%2fmath.stackexchange.com%2fquestions%2f3060107%2fis-there-a-way-to-sum-part-of-the-harmonic-series-to-a-given-total%23new-answer', 'question_page');
                      }
                      );

                      Post as a guest















                      Required, but never shown





















































                      Required, but never shown














                      Required, but never shown












                      Required, but never shown







                      Required, but never shown

































                      Required, but never shown














                      Required, but never shown












                      Required, but never shown







                      Required, but never shown







                      Popular posts from this blog

                      Quarter-circle Tiles

                      build a pushdown automaton that recognizes the reverse language of a given pushdown automaton?

                      Mont Emei